Finding Sup A for Set {0.2, 0.22, 0.222, 0.2222,...}

  • Thread starter nuuskur
  • Start date
  • Tags
    Set
In summary, the conversation discusses finding the supremum of a set A with elements that have repeating decimal values. The attempt at a solution involves proving the existence of a maximum value and showing that it does not exist. However, the speaker is unsure how to conclusively prove that the supremum does not exist. The conversation ends with a suggestion to consider a geometric series and making N arbitrarily large.
  • #1
nuuskur
Science Advisor
858
918

Homework Statement


Find sup A if A = {0.2, 0.22, 0.222, 0.2222, ...}
I'll write elements of a set with low case letters and indexes, e.g an

The Attempt at a Solution


Begin by definition of supremum:
[itex]\sup A = a[/itex] if [itex]\forall x \in A, x \leq a[/itex] and [itex]\forall b \in \mathbb R ((\forall x \in A , x \leq b) \rightarrow a \leq b[/itex] essentially, a supremum is the lowest value of upper bounds of a set.
Also if [itex]\exists \max A \rightarrow \sup A = \max A[/itex]

So I'll try to find a max A and prove sup A exists:
Let us have [itex]\exists \max A = M[/itex] such that [itex]\forall x \in A, x \leq M[/itex]
Suppose that [itex]a_n = M[/itex]. [itex]a_n = \sum_{k=0}^n\frac{1}{5 \cdot 10^k}, n \in \mathbb N[/itex]. If [itex]a_n \geq a_{n+1}[/itex] then [itex]\max A = \sup A = a_n[/itex]
[itex]a_{n+1}= \sum_{k=0}^{n}(\frac{1}{5 \cdot 10^n})+ \frac{1}{5 \cdot 10^{n+1}} \rightarrow a_{n+1} > a_n[/itex]. Therefore max A does not exist.

However, I cannot conclusively prove that sup A does not exist at all for this set A. Intuitively I can see that the elements' difference is becoming smaller and smaller and smaller, hence they should eventually be limited to some specific value. Altho it says if max A exists, it's also the sup A, but it does not say if max A doesn't exist, then there is no sup A.

What am I missing?

Thanks in advance.

EDIT:
Quick and dirty - If I assumed sup A = 0.23, for example, would it be sufficient evidence that sup A does not exist if I show there is a value 0.223 which is still greater than all the elements in the set A, but lesser than the supposed 0.23. Then it follows that I can suppose sup A = 0.222223, which still satisfies the upper bound criteria, however is not the least of the upper bounds. How can I show that there is no sup A in this case?
 
Last edited:
Physics news on Phys.org
  • #2
nuuskur said:

Homework Statement


Find sup A if A = {0.2, 0.22, 0.222, 0.2222, ...}
I'll write elements of a set with low case letters and indexes, e.g an


The Attempt at a Solution


Begin by definition of supremum:
[itex]\sup A = a[/itex] if [itex]\forall x \in A, x \leq a[/itex] and [itex]\forall b \in \mathbb R ((\forall x \in A , x \leq b) \rightarrow a \leq b[/itex] essentially, a supremum is the lowest value of upper bounds of a set.
Also if [itex]\exists \max A \rightarrow \sup A = \max A[/itex]

So I'll try to find a max A and prove sup A exists:
Let us have [itex]\exists \max A = M[/itex] such that [itex]\forall x \in A, x \leq M[/itex]
Suppose that [itex]a_n = M[/itex]. [itex]a_n = \sum_{k=0}^n\frac{1}{5 \cdot 10^k}, n \in \mathbb N[/itex]. If [itex]a_n \geq a_{n+1}[/itex] then [itex]\max A = \sup A = a_n[/itex]
[itex]a_{n+1}= \sum_{k=0}^{n}(\frac{1}{5 \cdot 10^n})+ \frac{1}{5 \cdot 10^{n+1}} \rightarrow a_{n+1} > a_n[/itex]. Therefore max A does not exist.

However, I cannot conclusively prove that sup A does not exist at all for this set A. Intuitively I can see that the elements' difference is becoming smaller and smaller and smaller, hence they should eventually be limited to some specific value. Altho it says if max A exists, it's also the sup A, but it does not say if max A doesn't exist, then there is no sup A.

What am I missing?

Thanks in advance.

EDIT:
Quick and dirty - If I assumed sup A = 0.23, for example, would it be sufficient evidence that sup A does not exist if I show there is a value 0.223 which is still greater than all the elements in the set A, but lesser than the supposed 0.23. Then it follows that I can suppose sup A = 0.222223, which still satisfies the upper bound criteria, however is not the least of the upper bounds. How can I show that there is no sup A in this case?

The least upper bound axiom states that if [itex]A \subset \mathbb{R}[/itex] is bounded above then it has a supremum [itex]M[/itex]. If it happens that [itex]M \in A[/itex] then [itex]M[/itex] is the maximum of [itex]A[/itex], but it may be that [itex]M \notin A[/itex] in which case [itex]A[/itex] has no maximum.

To your example: try summing the geometric series [tex]
a_N = \sum_{n=1}^N \frac{2}{10^n} = 2 \sum_{n=1}^N \frac{1}{10^n}[/tex] for fixed [itex]N[/itex]. Then consider what happens if you make [itex]N[/itex] arbitrarily large.
 
  • #3
Let x= 0.2222..., never ending.

Then 10x= 2.222..., still never ending.

Subtracting, 9x= 2.
 
  • #4
nuuskur said:

Homework Statement


Find sup A if A = {0.2, 0.22, 0.222, 0.2222, ...}
I'll write elements of a set with low case letters and indexes, e.g an


The Attempt at a Solution


Begin by definition of supremum:
[itex]\sup A = a[/itex] if [itex]\forall x \in A, x \leq a[/itex] and [itex]\forall b \in \mathbb R ((\forall x \in A , x \leq b) \rightarrow a \leq b[/itex] essentially, a supremum is the lowest value of upper bounds of a set.
Also if [itex]\exists \max A \rightarrow \sup A = \max A[/itex]

So I'll try to find a max A and prove sup A exists:
Let us have [itex]\exists \max A = M[/itex] such that [itex]\forall x \in A, x \leq M[/itex]
Suppose that [itex]a_n = M[/itex]. [itex]a_n = \sum_{k=0}^n\frac{1}{5 \cdot 10^k}, n \in \mathbb N[/itex]. If [itex]a_n \geq a_{n+1}[/itex] then [itex]\max A = \sup A = a_n[/itex]
[itex]a_{n+1}= \sum_{k=0}^{n}(\frac{1}{5 \cdot 10^n})+ \frac{1}{5 \cdot 10^{n+1}} \rightarrow a_{n+1} > a_n[/itex]. Therefore max A does not exist.

However, I cannot conclusively prove that sup A does not exist at all for this set A. Intuitively I can see that the elements' difference is becoming smaller and smaller and smaller, hence they should eventually be limited to some specific value. Altho it says if max A exists, it's also the sup A, but it does not say if max A doesn't exist, then there is no sup A.

What am I missing?

Thanks in advance.

EDIT:
Quick and dirty - If I assumed sup A = 0.23, for example, would it be sufficient evidence that sup A does not exist if I show there is a value 0.223 which is still greater than all the elements in the set A, but lesser than the supposed 0.23. Then it follows that I can suppose sup A = 0.222223, which still satisfies the upper bound criteria, however is not the least of the upper bounds. How can I show that there is no sup A in this case?


Of course there IS a sup in this case; I don't know why you think otherwise. In fact, it is a property of real numbers that any bounded set of real numbers has a supremum. However, maximum and supremum need not be the same thing. In this case the set has no maximum, but that does not matter for the problem at hand.

Others have already showed you how to find the supremum.
 

1. What is the value of Sup A for the given set?

The value of Sup A is 0.3.

2. How do you find the Sup A for a set of numbers?

To find the Sup A for a given set of numbers, you need to follow these steps:

  1. Arrange the numbers in ascending order
  2. Identify the largest number in the set
  3. Round up the number to the nearest hundredth
  4. The resulting number is the Sup A for the given set

3. What does Sup A stand for?

Sup A stands for the supremum or least upper bound of a set.

4. Why is it important to find the Sup A for a set?

The Sup A helps to determine the boundary or maximum limit of a set. It is useful in many mathematical and scientific calculations, such as finding the limit of a sequence or function.

5. Can the Sup A be a negative number?

No, the Sup A is always a positive number. It represents the largest number in a set and cannot be less than any of the numbers in the set.

Similar threads

  • Precalculus Mathematics Homework Help
Replies
15
Views
814
  • Set Theory, Logic, Probability, Statistics
Replies
3
Views
4K
  • Precalculus Mathematics Homework Help
Replies
15
Views
2K
Replies
1
Views
712
Replies
13
Views
1K
  • Calculus and Beyond Homework Help
Replies
14
Views
525
Replies
9
Views
890
  • Linear and Abstract Algebra
Replies
4
Views
1K
Replies
5
Views
389
Back
Top